Evidence MCs

Pataasin ang iyong marka sa homework at exams ngayon gamit ang Quizwiz!

A defendant was charged with murder, and a witness testified for the prosecution. On cross-examination of the witness, the defendant seeks to elicit an admission that the witness was also charged with the same murder and that the prosecutor told her, "If you testify against the defendant, we will drop the charges against you after the conclusion of the defendant's trial." The evidence about the prosecutor's promise is A: admissible, as proper impeachment of the witness. B: admissible, as an admission by an agent of a party-opponent. C: inadmissible, because the law encourages plea-bargaining. D: inadmissible, because the evidence is hearsay not within any exception.

A is correct. Evidence of bias or motive to lie is not hearsay and is admissible for impeachment purposes, which includes the prosecutor's promise to drop criminal charges against the witness, thus establishing a motive to provide testimony that is preferential to the prosecution.

A plaintiff sued an auto manufacturing company for his wife's death, claiming that a defective steering mechanism on the family car caused it to veer off the road and hit a tree when his wife was driving. The defendant claims that the steering mechanism was damaged in the collision and offers testimony that the deceased wife was intoxicated at the time of the accident. Testimony concerning the wife's intoxication is A: admissible to provide an alternate explanation of the accident's cause. B: admissible as proper evidence of the wife's character. C: inadmissible, because it is improper to prove character evidence by specific conduct. D: inadmissible, because it is substantially more prejudicial than probative

A is correct. Evidence of the plaintiff's wife's intoxication while she was driving is relevant and admissible to show the possible cause or causes of the accident. The evidence is admissible because the plaintiff is suing the defendant for products liability. It would be a valid defense if the wife was intoxicated at the time and thus caused the accident.

A corporation sued its former vice president for return of $230,000 that had been embezzled during the previous two years. Called by the corporation as an adverse witness, the vice president testified that his annual salary had been $75,000, and he denied the embezzlement. The corporation calls a banker to show that, during the two-year period, the vice president had deposited $250,000 in his bank account. The banker's testimony is A: admissible as circumstantial evidence of the vice president's guilt. B: admissible to impeach the vice president. C: inadmissible, because its prejudicial effect substantially outweighs its probative value. D: inadmissible, because the deposits could have come from legitimate sources.

A is correct. The banker's testimony regarding the vice president's deposits exceeding the amount that he legally earned has the tendency to make embezzlement more probable than it would be in the absence of such evidence.

In a medical malpractice suit by a patient against his doctor, the patient seeks to introduce a properly authenticated photocopy of the patient's hospital chart. The chart contained a notation made by a medical resident that an aortic clamp had broken during the plaintiff's surgery. The resident made the notation in the regular course of practice, but had no personal knowledge of the operation, and cannot remember which of the operating physicians gave him the information. The document is A: admissible as a record of regularly conducted activity. B: admissible as recorded recollection. C: inadmissible as a violation of the best evidence rule. D: inadmissible, because it is hearsay within hearsay.

A is correct. The chart is admissible as a "business record" because it is a record of acts, events, conditions, opinions, or diagnoses, made at or near the time by, or from information transmitted by, a person with knowledge, kept in the course of regularly conducted business, and it was the regular practice to make the record.

A defendant was prosecuted for bankruptcy fraud. The defendant's wife, now deceased, had testified adversely to the defendant during earlier bankruptcy proceedings that involved similar issues. Although the wife had been crossexamined, no serious effort was made to challenge her credibility despite the availability of significant impeachment information. At the fraud trial, the prosecutor offers into evidence the testimony given by the defendant's wife at the bankruptcy proceeding. This evidence should be A: admitted, under the hearsay exception for former testimony. B: admitted, because it is a statement by a person identified with a party. C: excluded, because it is hearsay not within any exception. D: excluded, because the defendant has the right to prevent use of his spouse's testimony against him in a criminal case.

A is correct. The defendant's wife's testimony at the prior hearing, if relevant to the bankruptcy fraud case, is admissible under the former testimony exception to the hearsay rule. Testimony given as a witness at another hearing of the same or a different proceeding, if the party against whom the testimony is now offered had an opportunity and similar motive to develop the testimony by direct, cross, or redirect examination, is admissible if the declarant is unavailable. In this case, the wife is now deceased and unavailable to testify. Although the testimony was in a separate proceeding, the defendant had the opportunity and motive to cross-examine his wife about the testimony, and the mere fact that he didn't use all he could have on cross-examination does not make the testimony inadmissible. Therefore, it is admissible under the former testimony exception.

A defendant is on trial for theft. At trial, the prosecutor called a husband and wife. They testified that, as they looked out their apartment window, they saw thieves across the street break the window of a jewelry store, take jewelry, and leave in a car. The wife telephoned the police and relayed to them the license number of the thieves' car as her husband looked out the window with binoculars and read it to her. Neither of them has any present memory of the number. The prosecutor offers as evidence a properly authenticated police tape recording of the wife's telephone call with her voice giving the license number, which is independently shown to belong to the defendant's car. The tape recording of the wife stating the license is A: admissible, under the hearsay exception for present sense impressions. B: admissible, as nonhearsay circumstantial evidence. C: inadmissible, because it is hearsay not within any exception. D: inadmissible, because the wife never had firsthand knowledge of the license number.

A is correct. The recording satisfies the present sense impression exception to the hearsay rule because the wife was relaying the information in real-time, as reported by the husband while he was observing the event live.

A famous author had a life insurance policy with an insurance company. Her son was the beneficiary. The author disappeared from her residence in a major city two years ago and has not been seen since. On the day that the author disappeared, a plane, which took off from the only airport in the city where the author lived, disappeared while flying over the ocean. The plane's passenger list included a passenger with the same first name as the author, but a different last name. The son is now suing the insurance company for the proceeds of his mother's policy. At trial, the son offers to testify that his mother told him that she planned to write her next novel under a pen name. The pen name she chose was the same name that appeared on the plane's passenger list. The son's testimony is A: admissible as circumstantial evidence that the author was on the plane. B: admissible as a party admission, because the author and her son are in privity with each other. C: inadmissible, because the author has not been missing more than seven years. D: inadmissible, because it is hearsay not within any exception.

A is correct. The son's testimony concerning his mother's statement that she planned to write her next novel under the pen name, which appeared on the plane's passenger list, is not hearsay and is admissible as circumstantial evidence that the author was on the plane. Although it is an out-of-court statement, the author's statement is not being offered for the truth of the matter asserted - that she planned to write her next novel using the pen name. Instead, the statement is being offered to show that the person on the plane under the pen name was, in fact, the author. Since the statement is not being offered for the truth of the matter asserted, it is admissible as relevant evidence.

In the prosecution of a defendant for murdering a victim, the defendant testified that the killing had occurred in selfdefense when the victim tried to shoot him. In rebuttal, the prosecution seeks to call a witness, the victim's father, to testify that the day before the killing, the victim told her father that she loved the defendant so much she could never hurt him. The witness's testimony is A: admissible within the hearsay exception for statements of the declarant's then existing state of mind. B: admissible, because the victim is unavailable as a witness. C: inadmissible as hearsay not within any exception. D: inadmissible, because the victim's character is not at issue

A is correct. The witness's testimony is being offered to show the victim's existing emotional and mental state when she made the statement and is thus admissible as an exception to the hearsay rule.

In a prosecution for aggravated battery, a police officer testified that when he arrested the defendant, he took a knife from the defendant and delivered it to the medical examiner. The medical examiner testified that the knife blade was consistent with the victim's wound but admitted on cross-examination that any number of other knives could also have caused the wound. Should the judge grant a motion to strike the medical examiner's testimony? A: No, because the probative worth of this evidence is for the jury to assess. B: Yes, because in light of the medical examiner's admission, his testimony has insufficient probative value. C: Yes, because the medical examiner could not state the probability that the wound was caused by the defendant's knife. D: Yes, because the probative value is substantially outweighed by the danger of unfair prejudice.

A is correct. This evidence has some probative value because it links the knife in the defendant's possession to the type of knife that could have caused the victim's wound. The evidence is not very strong, because other knives could also have caused the wound. But how much weight to give to the evidence is a decision for the jury. Rule 401 of the Federal Rules of Evidence (FRE) requires only that evidence have any tendency to make the existence of any fact of consequence more or less probable than it would be without the evidence. Thus, to be relevant, evidence need only have some probative value in establishing a fact.

At a defendant's murder trial, the defendant calls his first witness to testify that the defendant has a reputation in their community as a peaceable and truthful person. The prosecutor objects on the ground that the witness's testimony would constitute improper character evidence. The court should A: admit the testimony as to peaceableness, but exclude the testimony as to truthfulness. B: admit the testimony as to truthfulness, but exclude the testimony as to peaceableness. C: admit the testimony as to both character traits. D: exclude the testimony as to both character traits.

A is correct. Evidence of the defendant's reputation for peaceableness is relevant to the murder charge, a crime of violence, while his reputation for truthfulness is irrelevant to such a charge and no other basis exists to admit it. B is incorrect. The defendant's truthfulness is irrelevant here, whereas his reputation for peaceableness is relevant to a charge of murder and such evidence should be admitted. C is incorrect. While the testimony as to peaceableness should be admitted, the testimony regarding truthfulness should be excluded as impertinent. D is incorrect. For the reasons explained above, the evidence for truthfulness should be excluded and the evidence of peaceableness should be admitted.

In a suit by a plaintiff against a defendant, the plaintiff sought to subpoena an audiotape on which the defendant had narrated his version of the dispute for his attorney. Counsel for the defendant moves to quash the subpoena on the ground of privilege. The audiotape is most likely to be subject to subpoena if A: the defendant played the audiotape for his father to get his reactions. B: the lawsuit involved alleged criminal behavior by the defendant. C: the defendant has been deposed and there is good reason to believe that the audiotape may contain inconsistent statements. D: the defendant is deceased and thus unavailable to give testimony in person.

A is correct. If the defendant played the audiotape for his father to get his reaction, the privilege would be brokenbecause the communication is no longer confidential and the attorney-client privilege would not apply. B is incorrect. Unless the communication was used in furtherance of a crime or fraud known or reasonably shouldhave known by the client, alleged criminal activity will not break the privilege. C is incorrect. Whether or not the audiotape contains inconsistent statements is irrelevant to determining whetherthose statements are protected under the attorney-client privilege. D is incorrect. The attorney-client privilege extends beyond the death of the client.

A plaintiff sued a defendant for libel. After the plaintiff testified that the defendant wrote to the plaintiff's employer that the plaintiff was a thief, the defendant offers evidence that the plaintiff once stole money from a former employer. The evidence of the plaintiff's prior theft is A: admissible, as substantive evidence to prove that the plaintiff is a thief. B: admissible, but only to impeach the plaintiff's credibility. C: inadmissible, because character may not be shown by specific instances of conduct. D: inadmissible, because such evidence is more unfairly prejudicial than probative.

A is correct. One of the main issues in the case is whether or not the plaintiff is a thief (and consequently whether the defendant's letter to the employer was true), so the plaintiff's character trait for thievery is essential. Evidence that the plaintiff stole money from a former employer on a prior occasion is thus admissible to prove that the plaintiff is a thief. B is incorrect. The evidence of the plaintiff's prior theft can be used to prove the truth of the statement that the plaintiff is a thief, as well as for impeachment of the plaintiff's testimony. C is incorrect. The evidence that the plaintiff stole money demonstrates his character trait for thievery, which would bean essential element of the defendant's defense, and is substantively admissible. D is incorrect. The probative value of the evidence regarding the plaintiff's thievery is extremely high given that it is essential to the defense of truth. Moreover, any danger of unfair prejudice is minimal.

A defendant is on trial for tax evasion. The prosecutor, seeking to establish the defendant's income by showing his expenditures, has asked the defendant's attorney to produce records showing only how much the defendant had paid his attorney in fees. Should the demand for the attorney's fee records be upheld? A: Yes, because it calls for relevant information not within the attorney-client privilege. B: Yes, because the attorney-client privilege cannot be invoked to conceal evidence of a crime. C: No, because the records are protected by the attorney-client privilege. D: No, because the records are protected by the attorney work-product doctrine.

A is correct. The attorney-client privilege applies only to confidential communications made for the purpose of facilitating legal representation of the client, and the amount the defendant paid in legal fees does not qualify as such a communication. Fee arrangements and payments are generally outside the protection of the attorney-client privilege. B is incorrect. The attorney-client privilege can be invoked even where it conceals evidence of a past crime. C is incorrect. As stated above, fee arrangements and payments generally fall outside the protection of the attorney-client privilege. D is incorrect. The work product doctrine provides a qualified immunity for materials prepared by an attorney or client in anticipation of litigation, such as witness statements, investigative reports, or trial memoranda. The amount a client paid to his attorney for legal representation is outside the protection of the work-product doctrine.

A defendant has been charged with making a false statement to a federally insured financial institution to secure a loan. At trial, the prosecutor calls the defendant's wife as a willing witness to testify that the defendant told her in confidence that he had misrepresented his assets on the loan application. The defendant objects to his wife's testimony. Should the testimony be admitted? A: No, because even though the wife is a willing witness, the defendant has the right to exclude confidential marital communications in federal court. B: No, but only if the law of the state where the defendant and his wife reside recognizes a privilege for confidential marital communications. C: Yes, because in federal court the right not to testify belongs to the testifying spouse, and she is a willing witness. D: Yes, because while the adverse testimonial privilege is recognized in federal court, the marital communications privilege is not.

A is correct. The defendant may invoke the marital privilege because he told his wife the information in confidence, during their marriage, and no other facts are offered to indicate that the privilege would be otherwise inapplicable (such as abuse or a third party's presence). B is incorrect. Without any indication that a state law applies here to govern the applicability of the marital privilege, the federal rule should be applied. It is because the facts in this case meet the federal marital privilege rule that the defendant may exclude his wife's testimony. C is incorrect. Both spouses jointly hold the privilege, and either can refuse to disclose the communication or prevent the other from disclosing the communication. D is incorrect. The marital communications privilege is recognized by federal courts and in the Federal Rules ofEvidence.

In a civil trial arising from a car accident at an intersection, the plaintiff testified on direct examination that he came to a full stop at the intersection. On cross-examination, the defendant's lawyer asked whether the plaintiff claimed to have been exercising due care at the time, and the plaintiff replied that he had been driving carefully. At a sidebar conference, the defendant's lawyer sought permission to ask the plaintiff about two accidents in the previous 12 months in which he had received traffic citations for failing to stop at stop signs. The plaintiff's lawyer has objected. Should the court allow defense counsel to ask the plaintiff about the two prior accidents? A: No, because improperly failing to stop on the other occasions does not bear on the plaintiff's veracity and does not contradict his testimony in this case. B: No, because there is no indication that failing to stop on the other occasions led to convictions. C: Yes, because improperly failing to stop on the other occasions bears on the plaintiff's credibility, since he claims to have stopped in this case. D: Yes, because improperly failing to stop on the recent occasions tends to contradict the plaintiff's claim that he was driving carefully at the time he collided with the defendant.

A is correct. The prior accidents have no bearing on the plaintiff's character for truthfulness, which is required to be admissible under Federal Rule of Evidence (FRE) 608(b). Moreover, the prior accidents do not qualify as impeachment by contradiction, as they do not render the current testimony false. The defendant may not use the prior accidents merely to establish that the plaintiff is generally a careless driver. B is incorrect. A witness can be impeached with bad acts that do not result in convictions. Yet, the prior acts are inadmissible because they have no bearing on veracity under FRE 608(b) and are not contradictory to the current testimony. C is incorrect. Credibility is not affected by prior bad acts here, as they do not indicate untruthfulness. Rather, they are being offered merely to show careless driving, which is prohibited as improper character evidence under 404(a). D is incorrect. The prior bad acts that the plaintiff failed to stop do not contradict the current testimony that he did stop on this separate occasion.

At the start of the trial of a defendant and a codefendant for robbery, the codefendant and her attorney offered to give the prosecutor information about facts that would strengthen the prosecutor's case against the defendant in exchange for leniency toward the codefendant. The prosecutor refused the offer. Shortly thereafter, the codefendant committed suicide. During the defendant's trial, the prosecutor called the codefendant's attorney and asked him to relate the information that the codefendant had revealed to the attorney. Is the attorney's testimony admissible? A: No, because the codefendant's communications are protected by the attorney-client privilege. B: No, because the plea discussion was initiated by the codefendant rather than by the prosecutor. C: Yes, because the codefendant intended to disclose the information. D: Yes, because the information the codefendant gave to her attorney revealing her knowledge of the crime would be a statement against the codefendant's penal interest.

A is correct. The prosecutor is asking for confidential communications between the codefendant and her attorney, which is privileged information. If the codefendant had actually provided information to the prosecutor, the privilege would have been waived as to any communications previously made to her attorney. However, the codefendant did not disclose any confidential communications. B is incorrect. The prosecutor is asking for confidential communications between the codefendant and her attorney, which is privileged information, and it makes no difference who initiated the plea discussion. The question is whether the codefendant waived the privilege by offering information to the prosecutor, which she did not. Because the codefendant did not actually disclose any confidential communications to the prosecutor, there was no waiver. C is incorrect. The prosecutor is asking for confidential communications between the codefendant and her attorney, which is privileged information. If the codefendant had actually provided information to the prosecutor, the privilege would have been waived as to any communications previously made to her attorney. However, the codefendant did not disclose any confidential communications, and whether she intended to disclose the information is irrelevant. D is incorrect. A declaration against interest is one that tends to expose the declarant to criminal liability. Any statement to the attorney could not have subjected the codefendant to a risk of criminal liability, because the statement was privileged.

In a personal injury case, the plaintiff sued a retail store for injuries she sustained from a fall in the store. The plaintiff alleged that the store had negligently allowed its entryway to become slippery from snow tracked in from the sidewalk. Before the lawsuit was filed, when the plaintiff first threatened to sue, the store's manager said, "I know that there was slush on that marble entryway, but I think your four-inch-high heels were the real cause of your fall. So let's agree that we'll pay your medical bills, and you release us from any claims you might have." The plaintiff refused the offer. At trial, the plaintiff seeks to testify to the manager's statement that "there was slush on that marble entryway." Is the statement about the slush in the entryway admissible? A: No, because it is a statement made in the course of compromise negotiations. B: No, because the manager denied that the slippery condition was the cause of the plaintiff's fall. C: Yes, as a statement by an agent about a matter within the scope of his authority. D: Yes, because the rule excluding offers of compromise does not protect statements of fact made during compromise negotiations.

A is correct. The statement about the slush on the floor is inadmissible because it was part of a greater settlement negotiation excluded under FRE 408. B is incorrect. The defendant's denial that the slush was the cause of the fall is not the proper basis for excluding the statement. Rather, the protection of statements made during offers to compromise is the appropriate ground for exclusion. C is incorrect. Even if the statement satisfies an exemption to the hearsay rule, it may still be inadmissible under another Rule, as is the case here with FRE 408. D is incorrect. Offers to compromise are protected under FRE 408, but so are "conduct or statements made in the course of compromise negotiations."

Dryden is tried on a charge of driving while intoxicated. When Dryden was booked at the police station, a videotape was made that showed him unsteady, abusive, and speaking in a slurred manner. If offered by the prosecution as evidence of Dryden's intoxication, the trial court should: A: Admit the evidence because it is not hearsay. B: Exclude the evidence because it is assertive conduct. C: Admit the evidence because although hearsay, it is an admission of a party opponent. D: Exclude the evidence because its admission would violate Dryden's confrontation clause rights.

A: Admit the evidence because it is not hearsay.

One day, while driving home from work in her Toyota Camry, Carla is involved in a car accident in which her airbag fails to deploy. As a result, Carla is severely injured. Carla sues both the other driver involved in the accident and Toyota. At trial, during the defendant's case-in-chief, the lead design engineer for the Toyota Camry takes the stand to testify regarding the design of the airbag. On cross-examination, the plaintiff's attorney asks the engineer to describe the deployment system of the airbag. The engineer describes the mechanism that allows the airbag to detect a collision and the process by which the airbag is deployed. The engineer further states that this has been the same mechanism used in the Toyota Camry since 2006, and that no changes have been made to the Camry's deployment mechanism since 2006. The plaintiff's attorney then asks the engineer whether Toyota, in fact, changed the design of the airbag deployment system after the accident in 2015. Defense counsel objects. How should the court rule? A: OVERRULE the objection, because the question is being used for impeachment purposes B: SUSTAIN the objection, because the evidence is irrelevant C: OVERRULE the objection, because the feasibility of the repair was not contested D: SUSTAIN the objection, because the question calls for evidence regarding subsequent remedial measures

A: OVERRULE the objection, because the question is being used for impeachment purposes

Fracme Energy Company was exploring for oil using a technique called hydraulic fracturing, or "fracking." Monica owned a business nearby. Monica sued Fracme, alleging that Fracme's fracking activities caused tremors in the ground that damaged her building. Monica hired a scientist named Enrique to measure the tremors with a seismograph, which is an instrument that measures and records details of tremors and earthquakes. At the trial, Monica's attorney called Enrique as a witness and asked him, "On June 12 at 1:00 p.m., what did the seismograph say?" To prove that there was a tremor of magnitude 4.0 at that time, Enrique planned to testify: "The seismograph said that there was a tremor of magnitude 4.0 at that time." Fracme's attorney replied, "Objection. . . calls for hearsay." How should the judge rule on the objection? A: OVERRULE the objection, because the statement is not hearsay. B: SUSTAIN the objection, because Enrique is testifying as to an out-of-court statement C: SUSTAIN the objection, because too much time has elapsed for the present-sense-impression to apply D: OVERRULE the objection, because the present-sense-impression exception applies

A: OVERRULE the objection, because the statement is not hearsay.

Joseph Jones sues Carla Carter for breach of contract. Carter asserts that she was unable to collect funds from her client, Will Worthingham, in order to satisfy the contract. Worthingham is not a party to the case. Carter seeks to introduce Worthingham's statement to Carter that Worthingham owed his creditors over $300,000. Worthingham is available to testify at trial. Joseph objects to the statement as hearsay. Assuming that Worthingham's statement is relevant, how should the judge rule on the objection? A: SUSTAIN the objection because the declarant is available to testify. B: OVERRULE the objection as a statement against interest. C: SUSTAIN the objection because the statement is not against the interest of Carter. D: SUSTAIN the objection because the statement is not against the interest of Jones.

A: SUSTAIN the objection because the declarant is available to testify.

On May 1 at the corner of First Avenue and Main Street, Stan shot Oleg, who died as a result. Right before the shooting, an eyewitness named Elizabeth standing at Second Avenue and Main Street saw Stan walking down the street carrying a large knife with a red handle. On July 1, Elizabeth was recounting the crime to her husband, Phillip, and she told him that she had seen someone walking down the street carrying a large knife with a red handle. Stan was charged with murder, and his trial began on November 1. At Stan's trial, the prosecution called Elizabeth as a witness to testify as to what she had seen. On cross-examination, Stan's attorney asked Elizabeth, "Isn't it true that on June 1, someone from the prosecutor's office offered you a bribe in exchange for testifying the way you just did?" The prosecutor later calls Phillip and offers his testimony that on July 1, Elizabeth told him that she had seen someone walking down the street carrying a large knife with a red handle. Stan's attorney objects based on hearsay. How should the judge rule on this objection? A: SUSTAIN the objection, because Elizabeth made her out-of-court statement after the alleged bribe B: OVERRULE the objection, because the prosecution could properly use Phillip's testimony about Elizabeth's statement to rebut Stan's express charge that Elizabeth acted from a recent improper influence C: SUSTAIN the objection, because Elizabeth did not make her out-of-court statement under penalty of perjury at a trial, hearing, or other proceeding or in a deposition D: OVERRULE the objection, because Elizabeth's out-of-court statement to Phillip consisted of words of independent legal significance or was a verbal act

A: SUSTAIN the objection, because Elizabeth made her out-of-court statement after the alleged bribe

Susan threw a huge party at her apartment and invited every student from her first-year section of law school. Everyone was having a great time, although several party-goers had too much to drink. One such attendee tripped over the threshold of Susan's apartment door, fell on his face, and broke his nose. The student sued Susan, asserting that the threshold was a dangerous condition and hadn't been nailed down properly. Susan's sole defense is that the student was contributorily negligent because he was intoxicated. The student seeks to introduce evidence that, after the party, Susan nailed down the threshold properly, solely to prove that Susan exerted control over the entryway to her apartment. Susan objects. Is the evidence admissible? A: The evidence is inadmissible under the facts described. B: The evidence is admissible only to show the repair was feasible. C: The evidence is admissible if its prejudicial effect is substantially outweighed by its probative value. D:The evidence is admissible to show that Susan exerted control over the threshold to the apartment.

A: The evidence is inadmissible under the facts described.

Goldberg sues Dangerfield, claiming that on October 1 Dangerfield fraudulently sold Goldberg a barren cow. To prove that Dangerfield knew the cow was barren, Goldberg offers the testimony of Max that, "I was with Dangerfield when the vet told him that the cow was barren." Which of the following is the most accurate? A: The statement is admissible because it is not hearsay. B: The statement is inadmissible hearsay. C: The statement is admissible as a party admission. D: The statement is admissible as a statement for purposes of medical treatment.

A: The statement is admissible because it is not hearsay.

Gilfoyle was charged with robbing a bank. A week after the robbery, a witness named Dinesh picked Gilfoyle out of a photographic array and told Detective Bachman that Gilfoyle was the bank robber. A month later, the police caught Gilfoyle and asked Dinesh to identify him from an in-person lineup. During this lineup, Dinesh was very uncertain as to the identity of the robber, and he ended up identifying a police-department employee who was merely filling out the lineup but had nothing to do with the robbery. As a result, the prosecution decided not to call Dinesh as a witness, and Dinesh did not attend the trial. At Gilfoyle's trial, the prosecution calls Detective Bachman and seeks to offer his testimony that Dinesh picked Gilfoyle out of the original photographic array and identified Gilfoyle as the bank robber. Gilfoyle objects. Is Dinesh's statement to Detective Bachman hearsay? A: YES, and the exclusion for a statement identifying a person as someone the declarant perceived earlier does not apply B: NO, because Dinesh's statement is inconsistent with his statement at the in-person lineup and is therefore excluded from being hearsay C: NO, because Dinesh's statement identifies a person as someone he perceived earlier and is therefore excluded from being hearsay D: YES, but it is admissible under the exception for former testimony

A: YES, and the exclusion for a statement identifying a person as someone the declarant perceived earlier does not apply

The defendant was charged with stealing furs from a van. At trial, a witness testified that she saw the defendant take the furs. The jurisdiction in which the defendant is being tried does not allow in evidence lie detector results. On crossexamination by the defendant's attorney, the witness was asked, "The light was too dim to identify the defendant, wasn't it?" She responded, "I'm sure enough that it was the defendant that I passed a lie detector test administered by the police." The defendant's attorney immediately objects and moves to strike. The trial court should A: grant the motion, because the question was leading. B: grant the motion, because the probative value of the unresponsive testimony is substantially outweighed by the danger of unfair prejudice. C: deny the motion, because it is proper rehabilitation of an impeached witness. D: deny the motion, because the defendant's attorney "opened the door" by asking the question.

B is correct. Although relevant, evidence may be excluded if its probative value is substantially outweighed by the danger of unfair prejudice. Because lie detector results are inadmissible in this jurisdiction, the trial court should grant the motion to strike the testimony referring to inadmissible evidence.

A defendant was prosecuted for the murder of a victim, whose body was found one morning in the street near the defendant's house. The state calls a witness, a neighbor, to testify that during the night before the body was found he heard the defendant's wife scream, "You killed him! You killed him!" The witness's testimony is A: admissible, as a report of a statement of belief. B: admissible, as a report of an excited utterance. C: inadmissible, because it reports a privileged spousal communication. D: inadmissible on spousal immunity grounds, but only if the wife objects.

B is correct. The defendant's wife's statement is an out-of-court statement that is being offered for the truth of the matter asserted - that the defendant killed the victim - so it is hearsay. However, the statement related to a startling event or condition made while the wife was under the stress of excitement caused by the event or condition, and is thus admissible as an excited utterance. Therefore, the witness's testimony is admissible.

A man and his friend were charged with conspiracy to dispose of a stolen diamond necklace. The friend jumped bail and cannot be found. Proceeding to trial against the man alone, the prosecutor calls the friend's girlfriend as a witness to testify that the friend confided to her that "[the man] said I still owe him some of the money from selling that necklace." The witness's testimony is A: admissible, as evidence of a statement by party-opponent. B: admissible, as evidence of a statement against interest by the friend. C: inadmissible, because the friend's statement was not in furtherance of the conspiracy. D: inadmissible, because the friend is not shown to have firsthand knowledge that the necklace was stolen.

B is correct. The testimony contains two separate hearsay statements, both of which must satisfy an exemption or exception for the testimony to be admissible. The statement by the man to the friend is an admission by a partyopponent, and is therefore exempt as non-hearsay. The statement by the friend to the girlfriend constitutes an exception as a statement against interest. Therefore, the testimony is admissible.

In a civil action for breach of an oral contract, the defendant denied having entered into a contract with the plaintiff, although he admitted that he had discussed doing so. Which of the following standards of admissibility should the court apply to evidence proffered as relevant to prove whether a contract was formed? A: Whether a reasonable juror would find the evidence determinative of whether the contract was or was not formed. B: Whether the evidence has any tendency to make the fact of contract formation more or less probable than without the evidence. C: Whether the evidence is sufficient to prove, absent contrary evidence, that the contract was or was not formed. D: Whether the evidence makes it more likely than not that a contract was or was not formed.

B is correct. This is the correct standard for relevance to be applied by the judge in determining admissibility under Federal Rule of Evidence (FRE) 401.

A defendant was charged with attempted murder of a victim in a sniping incident in which the defendant allegedly shot at the victim from a bush as the victim drove his car along an expressway. The prosecutor offers evidence that seven years earlier the defendant had fired a shotgun into a woman's house and that the defendant had once pointed a handgun at another driver while driving on the street. This evidence should be A: excluded, because such evidence can be elicited only during cross-examination. B: excluded, because it is improper character evidence. C: admitted as evidence of the defendant's propensity toward violence. D: admitted as relevant evidence of the defendant's identity, plan, or motive.

B is correct. Evidence of "prior bad acts" is inadmissible against a defendant when offered to prove that he is more likely to have committed the instant crime based on criminal propensity. A is incorrect. Not only has the defendant not testified, but even if he were to testify, this evidence cannot be raised on cross-examination because specific incidents are only admissible when they are probative of truthfulness or untruthfulness. C is incorrect. Evidence of other crimes, wrongs, or acts is not admissible character evidence when it is used in order to show action in conformity with those wrongs. D is incorrect. There is no indication that the prosecution is seeking to introduce the evidence to prove identity, plan, or motive, rather, it is merely being used to establish the defendant's criminal propensity, which is an improper use of character evidence.

A defendant was charged with using a forged prescription from a doctor to obtain Percodan from a drugstore on May 1. At trial, the drugstore owner identified the defendant as the customer, but the defendant testified that he had not been in the store. In rebuttal, the prosecutor calls two additional drug store owners to testify that, on May 1, a man they identified as the defendant had presented prescriptions for Percodan from a doctor at their drug stores. The additional drug store owners' testimony is A: admissible, to prove a pertinent trait of the defendant's character and the defendant's action in conformity therewith. B: admissible, to identify the man who presented the prescription at the original drugstore. C: inadmissible, because it proves specific acts rather than reputation or opinion. D: inadmissible, because other crimes may not be used to show propensity.

B is correct. Evidence of other crimes, wrongs, or acts is not admissible to prove the character of a person in order to show action in conformity therewith. However, other crimes evidence can be admitted to show identity, intent, preparation, or plan. The evidence the prosecutor seeks to admit is that the exact same man, the defendant, had presented a forged prescription for the exact same drug, on the exact same day. That evidence would be admissible to prove the identity, which the defendant called into question by his testimony, of the person who obtained the Percodan at the drugstore. A is incorrect. Evidence of other crimes, wrongs, or acts is not admissible to prove the character of a person in order to show action in conformity therewith. C is incorrect. The additional drug store owners' testimony is not being used to prove character; it is being used to prove identity. Used for this purpose, the testimony is admissible even though it covers specific acts. D is incorrect. Although evidence of other crimes is inadmissible to prove propensity, evidence of the other crimes in this case is admissible to prove identity.

At a civil trial for slander, the plaintiff showed that the defendant had called the plaintiff a thief. In defense, the defendant called a witness to testify, "I have been the plaintiff's neighbor for many years, and people in our community generally have said that he is a thief." Is the testimony concerning the plaintiff's reputation in the community admissible? A: No, because character is an essential element of the defense, and proof must be made by specific instances of conduct. B: Yes, to prove that the plaintiff is a thief, and to reduce or refute the damages claimed. C: Yes, to prove that the plaintiff is a thief, but not on the issue of damages. D: Yes, to reduce or refute the damages claimed, but not to prove that the plaintiff is a thief.

B is correct. The nature of the plaintiff's character is directly tied to the defendant's defense that his statement (that the plaintiff was a thief) was true. Furthermore, the plaintiff's character is relevant to the amount of damages to which he is entitled. A is incorrect. Proof of character evidence may be introduced in the form of opinion or reputation testimony, not only specific acts. C is incorrect. Character evidence is also admissible for establishing the level of damages done (or not done) to the plaintiff's reputation. D is incorrect. Character evidence is also admissible to prove that the plaintiff is a thief, an essential element of the case.

A defendant has pleaded not guilty to a federal bank robbery charge. The principal issue at trial is the identity of the robber. The prosecutor has called the defendant's wife to testify to the clothing that the defendant wore as he left their house on the day the bank was robbed, expecting her description to match that of eyewitnesses to the robbery. Both the defendant and his wife have objected to her testifying against the defendant. Should the wife be required to testify? A: No, because the defendant has a privilege to prevent his wife from testifying against him in a criminal case. B: No, because the wife has a privilege not to testify against her husband in a criminal case. C: Yes, because the spousal testimonial privilege does not apply in criminal cases. D: Yes, because the wife's viewing of the defendant's clothing was not a confidential communication.

B is correct. This is a correct statement of federal common law, established by the Supreme Court in Trammel v. United States. If the witness and the defendant are married at the time of trial, the witness cannot be placed in contempt for refusing to testify against the defendant. The rationale for the rule is to preserve marital harmony, which would otherwise be damaged by one spouse testifying against the other. A is incorrect. Under federal common law, the spousal testimonial privilege is held by the witness, not by the defendant. If a spouse wants to testify, the defendant cannot prevent the spouse from doing so. The rationale is that the privilege is designed to preserve marital harmony, and if a witness wants to testify against his or her spouse, then there is no marital harmony left to preserve. C is incorrect. The rule is in fact the opposite. Under federal common law, the interspousal testimonial privilege applies only in criminal cases and not in civil cases. D is incorrect. This statement confuses two different privileges under federal common law. The interspousal communications privilege protects confidential communications made between the spouses during the marriage. That privilege is not at issue in this fact situation because testimony about the defendant's appearance at the time of the crime would not disclose a confidential communication. But there is also a testimonial privilege allowing a spouse to refuse to testify against his or her spouse, regardless of whether the testimony would involve confidential communications. It is this testimonial privilege that is applicable in the present fact situation.

A woman brought a products liability action against a drug manufacturer in federal court under state law, claiming that the manufacturer had failed to warn that an arthritis drug caused severe hair loss in female patients. In discovery, the manufacturer inadvertently turned over to the woman two documents reflecting communications between the manufacturer's president and its counsel regarding the drug's possible side effects. There were 23 other, similar documents reflecting such communications that were not turned over in discovery. Although the manufacturer learned of the disclosure during the discovery period, it did not seek the return of the two documents until the day before trial. In response, the woman claimed that the manufacturer had waived the attorney client privilege as to all 25 documents, including the 23 that had not been turned over. How should the court rule on the woman's claim of waiver of privilege? A: There was no waiver of the attorney-client privilege, because the disclosure to the woman was inadvertent. B: There was a waiver of the attorney-client privilege regarding only the two disclosed documents. C: There was a waiver of the attorney-client privilege regarding the disclosed documents as well as the other 23 documents, because they all relate to the same subject matter. D: There was a waiver of the attorney-client privilege for all 25 documents if the state law that supplies the rule of decision would support such a result.

B is correct. Under Federal Rule Evidence (FRE) 502(a), a waiver of privilege extends to an undisclosed communication or information only if the waiver is intentional; the disclosed or undisclosed communications or information concern the same subject matter; and they ought in fairness be considered together. Here, because the disclosure was inadvertent and not intentional, the waiver does not extend to the undisclosed communications. A is incorrect. Under FRE 502(b), an inadvertent disclosure will not operate as a waiver of the attorney-client privilege if the holder of the privilege "promptly [takes] reasonable steps to rectify the error." Here, the manufacturer learned of the disclosure during discovery but waited until the day before trial to rectify the error. Its response was not "prompt," and therefore the disclosure operates as a waiver as to the two documents that were disclosed. C is incorrect. Under FRE 502(a), because the disclosure was inadvertent, the waiver does not extend to undisclosed communications (the other 23 documents) even if they concern the same subject matter. D is incorrect. FRE 502 applies to "disclosure[s] . . . made in a federal proceeding." Thus, state law does not apply, even when federal jurisdiction is based on diversity of citizenship.

A plaintiff sued a defendant in connection with the dissolution of a partnership they had formed to run a parcel delivery service. They had relied on a business attorney in establishing the business. After the business failed, the plaintiff and the defendant disagreed about their respective obligations. At trial, both have hired new counsel. The plaintiff calls the business attorney to testify to representations the defendant made in meetings she had with the plaintiff and the business attorney. The defendant objects to the business attorney's testimony, invoking the attorney-client privilege. Should the court uphold the defendant's privilege claim? A: No, because the business attorney's professional relationship with the plaintiff and the defendant has ended. B: No, because the plaintiff and the defendant consulted the business attorney jointly. C: Yes, because either the plaintiff or the defendant may block disclosure of statements made during such meetings. D: Yes, because either the plaintiff or the defendant may claim the privilege on behalf of the partnership.

B is correct. When the attorney-client privilege is invoked by a client who was jointly represented along with another client originally, and the clients enter into a subsequent dispute, as is the case here, the privilege no longer applies. A is incorrect. The privilege has not been extinguished simply because the professional relationship has ended (the privilege still stands as to outsiders). The privilege merely does not extend to litigation between the two joint clients. C is incorrect. As stated above, when an attorney jointly represents two people, the privilege does not extend to subsequent litigation between the parties, and thus neither the plaintiff nor the defendant may block disclosure of the statements. D is incorrect. In this situation, the jointly-represented clients no longer have the privilege as applied to the partnership because one client's interest in the privilege is counterbalanced by the other's interest in being able to waive it.

Theresa Grady, Peter Dumont, Daniel Burns, and Colleen First, the St. Patrick's Day 4, were arrested and tried for destruction of property. The four entered a United States army recruiting center on March 17, got on their knees, prayed for peace and poured samples of their own blood onto the floor and desks as protest against the Iraq war which they argued was illegal. The state government offered the testimony of Sergeant Raul Rodriguez, who was present at the center and witnessed the demonstration. At the state-court level, the St. Patrick's Day 4 were all acquitted. The four were indicted on federal conspiracy charges of conspiracy to intimidate a federal officer based upon the same facts. At the federal level, if the federal government offers a transcript of Sergeant Rodriguez's earlier state court testimony at the federal trial, the evidence should be: A: ADMISSIBLE, provided that Sergeant Rodriguez is available at the federal trial to be cross examined concerning his earlier testimony B: ADMISSIBLE, if Sergeant Rodriguez is unavailable to testify at the federal trial C: INADMISSIBLE, because to allow the transcript in would violate the Confrontation Clause D: ADMISSIBLE, provided that Sergeant Rodriguez was sworn to tell the truth whether he was cross examined or not

B: ADMISSIBLE, if Sergeant Rodriguez is unavailable to testify at the federal trial

Dave is on trial for the hate-crime murder of Khalid. The government's theory is that Dave gave Frank a gun and told Frank to kill Khalid and that Frank then tracked down Khalid and shot him to death. In order to prove that Frank killed Khalid, the government offers evidence that on the day before Khalid was found dead for a gunshot wound, Frank told his friend Wally, "Tomorrow I'm going to find Khalid where he works and blow his head off." Frank is available to testify at trial, and the government wants to introduce evidence of his out-of-court statement to Wally. Evidence of Frank's statement to Wally is: A: Does not meet the definition of hearsay. B: Hearsay but admissible as a state-of-mind exception to the hearsay rule. C: Inadmissible hearsay. D: Hearsay but admissible under the statement-against-interest exception.

B: Hearsay but admissible as a state-of-mind exception to the hearsay rule.

James and Mike were stockbrokers who worked together at the same brokerage firm. James is now on trial for insider trading. The prosecutor wants to admit statements made by Mike which implicate James in the insider trading scheme. James objects, arguing that Mike's statements are hearsay. The prosecutor responds that although Mike's statements are hearsay, they should be admitted as an opposing party's statement because Mike and James were co-conspirators. The trial judge rules that Mike and James were co-conspirators and admits the evidence. James appeals the trial court's decision to admit Mike's statements. How should the appellate court rule? A: Overturn the lower court because the jury, not the judge, should decide whether a conspiracy existed. B: Overturn the lower court ruling if the ruling was an abuse of discretion and affected James' substantial rights C: Conduct a de novo review of whether the evidence should have been admitted and overturn the trial court if the evidence should have been excluded. D: Overturn the lower court if no reasonable juror could find that a conspiracy existed.

B: Overturn the lower court ruling if the ruling was an abuse of discretion and affected James' substantial rights

Darcy Davis was charged with aggravated assault by a grand jury. Darcy got into an altercation with Susan Silver at Minute Maid Park. Darcy picked up the baseball bat that she had received as being one of the first 200 entrants to the park that day and repeatedly hit Susan over the head with it. Susan was badly injured and bleeding, and when the park's security guards came to her assistance she stated "I can't believe Darcy did this to me! Now I'm never going to see my kids grow up." Susan then passed out and was taken to the hospital, where she made a complete recovery. After the trauma of the assault, Susan left town and the prosecution, despite diligent efforts, cannot locate her. The prosecution seeks to introduce Susan's statement at trial. Darcy objects, arguing hearsay. The prosecution argues this falls within the dying declaration exception. How should the trial judge rule on the objection? A: SUSTAIN the objection because Susan did not die. B: SUSTAIN the objection because the dying declaration exception is inapplicable. C: SUSTAIN the objection because, although the dying declaration exception applies, the statement is inadmissible under the confrontation clause. D: OVERRULE the objection under the dying declaration exception.

B: SUSTAIN the objection because the dying declaration exception is inapplicable.

Under the rule allowing exclusion of relevant evidence because its probative value is substantially outweighed by other considerations, which of the following is NOT to be considered? A: The jury may be confused about the appropriate application of the evidence to the issues of the case. B: The evidence is likely to arouse unfair prejudice on the part of the jury. C: The opponent is surprised by the evidence and not fairly prepared to meet it. D: The trial will be extended and made cumbersome by hearing evidence of relatively trivial consequence

C is correct. Relevant evidence may be excluded if its probative value is substantially outweighed by the danger of unfair prejudice, confusion of the issues, misleading the jury, or undue delay, waste of time, or needless presentation of cumulative evidence. Only this answer does not deal with any of those considerations and instead deals with the exclusion of evidence-based on potential discovery violations.

A defendant's house was destroyed by fire, and she was charged with arson. To prove that the defendant had a motive to burn down her house, the government offered evidence that the defendant had fully insured the house and its contents. Should the court admit this evidence? A: No, because the probative value of the evidence of insurance on the issue of whether the defendant intentionally burned her house down is substantially outweighed by the dangers of unfair prejudice and confusion of the jury. B: No, because evidence of insurance is not admissible on the issue of whether the insured acted wrongfully. C: Yes, because evidence of insurance on the house has a tendency to show that the defendant had a motive to burn down the house. D: Yes, because any conduct of a party to the case is admissible when offered against the party.

C is correct. The evidence is relevant to the issue of the defendant's incentive to commit the arson, and its probative value is not substantially outweighed by the danger of unfair prejudice, confusion of the issues, undue delay, etc

A plaintiff sued for injuries arising from a car accident, claiming a back injury. At trial, she wishes to testify that before the accident she had never had any problems with her back. Is the plaintiff's proposed testimony admissible? A: No, because the plaintiff has not been qualified as an expert. B: No, because the plaintiff's pain could have been caused by factors arising after the accident, such as an injury at work. C: Yes, because it is probative evidence of the plaintiff's injury. D: Yes, because the testimony of parties is not subject to the lay opinion rule.

C is correct. The fact that the back problems arose after the accident is probative on the issue of whether the accident was the cause of the injury. Therefore, the evidence meets the standard of relevance, and the testimony should be allowed.

In a prosecution of a defendant for assault, a witness is called to testify that the victim had complained to the witness that the defendant was the assailant. The witness's testimony is most likely to be admitted if the witness is A: a doctor, whom the victim consulted for treatment. B: a minister, whom the victim consulted for counseling. C: the victim's husband, whom she telephoned immediately after the event. D: a police officer, whom the victim called on instructions from her husband.

C is correct. The most likely basis for admitting the witness's testimony is if the victim called her husband immediately after the event because those facts would satisfy the excited utterance exception to the hearsay rule.

A defendant was prosecuted in federal court for making threats against the President of the United States. The defendant was a voluntary patient in a private psychiatric hospital and told a nurse, shortly before the President came to town, that the defendant planned to shoot the President. The nurse reported the threat to FBI agents. The defendant's motion to prevent the nurse's testifying is likely to be A: successful, because the statement was made in a medical setting. B: successful, because the nurse violated a confidence in reporting the statement. C: unsuccessful, because the statement was not within any privilege. D: unsuccessful, because the defendant had not been committed involuntarily by court order.

C is correct. Although federal courts do recognize a psychiatrist-patient privilege, the defendant's statement to the nurse that he planned to shoot the President was not confidential and was not made to obtain treatment. Therefore, the statement is unprivileged and the nurse should be allowed to testify about the statement. A is incorrect. Although the statement was made in a medical setting, the privilege only covers confidentialcommunications between a licensed psychotherapist and her patients in the course of diagnosis or treatment. Not every statement in a medical setting is privileged, and the defendant's statement to the nurse was not a confidential communication made in the course of diagnosis or treatment. B is incorrect. Whether or not the statement was made in confidence is not the legal standard to determine itsadmissibility. The statement was not subject to the privilege. D is incorrect. Whether the defendant was there voluntarily or committed involuntarily has no relevance to the determination of whether the privilege applies.

A defendant is on trial for arson. In its case in chief, the prosecution offers evidence that the defendant had secretly obtained duplicate insurance from two companies on the property that burned and that the defendant had threatened to kill his ex-wife if she testified for the prosecution. The court should admit evidence of A: the defendant's obtaining duplicate insurance only. B: the defendant's threatening to kill his ex-wife only. C: both the defendant's obtaining duplicate insurance and threatening to kill his ex-wife. D: neither the defendant's obtaining duplicate insurance nor threatening to kill his ex-wife.

C is correct. Both the defendant's obtaining duplicate insurance and the threat to kill his ex-wife if she testifies are relevant and admissible under FRE 404(b), for purposes other than bad character to show conformity therewith. The duplicate insurance is relevant to establishing the defendant's motive for committing arson. The threat to kill his ex-wife is relevant as to his plan, knowledge, and consciousness of guilt. A is incorrect. Evidence of the defendant's obtaining duplicate insurance is admissible, but the threat to kill his ex-wife is also admissible to establish plan, knowledge, and consciousness of guilt. B is incorrect. Evidence of the defendant's threat to kill his ex-wife is admissible, but obtaining duplicate insurance is also admissible to establish motive. D is incorrect. As explained above, both the duplicate insurance and the threat to kill his ex-wife are admissible.

A lawyer was called to testify before a grand jury that was investigating a hit-and-run accident. A security guard at the lawyer's office had testified earlier that on the date of the accident he had seen a person leave the lawyer's office at around 7 p.m., get into a car, and strike a pedestrian while driving out of the parking lot without stopping afterward. The lawyer was asked to disclose the identity of the person who left his office around 7 p.m. on the date of the accident. The lawyer stated that the person was a client but refused to disclose the client's identity, citing the attorney client privilege. Can the lawyer be compelled to disclose the client's identity to the grand jury? A: No, because the identity of a lawyer's client is protected by the attorney-client privilege. B: No, because the lawyer's testimony would undermine the client's privilege against self-incrimination. C: Yes, because the identity of a lawyer's client is not privileged when disclosure would not reveal a confidential attorney-client communication. D: Yes, because the rules of evidence do not apply at a grand jury proceeding.

C is correct. Disclosing the client's identity to the grand jury would not reveal anything about the communications between the client and the lawyer. Therefore, the attorney-client privilege does not apply, and the lawyer can be compelled to make the disclosure. A is incorrect. Generally, the identity of a lawyer's client is not protected by the attorney-client privilege because such a disclosure would not reveal any confidential attorney-client communications. B is incorrect. The privilege against self-incrimination does not prevent the lawyer from disclosing the client's identity.Rather, it prevents the client from being compelled to give harmful testimony. D is incorrect. Under Federal Rule of Evidence 1101(c), rules on privilege apply to all stages of a case, including grand jury proceedings.

A defendant is on trial for the murder of his father. The defendant's defense is that he shot his father accidentally. The prosecutor calls a witness, a police officer, to testify that on two occasions in the year prior to this incident, he had been called to the defendant's home because of complaints of loud arguments between the defendant and his father, and had found it necessary to stop the defendant from beating his father. The evidence is A: inadmissible, because it is improper character evidence. B: inadmissible, because the witness lacks first-hand knowledge of who started the quarrels. C: admissible to show that the defendant killed his father intentionally. D: admissible to show that the defendant is a violent person.

C is correct. Evidence of the prior fights and beatings is admissible for the purpose of establishing the defendant's intent or absence of mistake, given that his defense is that he unintentionally shot his father. A is incorrect. This type of character evidence is not being offered to prove action in conformity therewith, but rather intent and absence of mistake or accident. B is incorrect. The witness does not need to know who started the quarrels for the evidence to be relevant to the issue of establishing intent or absence of mistake or accident. D is incorrect. While prior bad acts are not admissible to prove one's violent character, they are admissible to show intent and lack of mistake or accident.

A woman sued her friend for injuries she received as a passenger in the friend's car. On direct examination, the woman testified that the friend had been speeding and ran a red light. On cross-examination, the woman was asked whether she was under the influence of drugs at the time of the accident. The woman invoked the privilege against self-incrimination. How should the court treat the woman's claim of privilege? A: Deny it, because the woman waived the privilege by voluntarily testifying. B: Deny it, because evidence of the woman's drug intoxication is essential to assessing the accuracy of her observations. C: Uphold it, because the privilege applies in both civil and criminal cases. D: Uphold it, because the woman's credibility cannot be impeached by a crime for which she has not been convicted.

C is correct. If a witness's testimony in a civil (or criminal) proceeding would subject her to potential criminal liability she may properly invoke the right against self-incrimination. A is incorrect. A witness may waive the privilege against self-incrimination by voluntarily testifying. Here, however, the woman had not yet testified about incriminating matters. B is incorrect. Although the woman's possible impairment would be proper grounds for impeaching her testimony, her constitutional privilege against self-incrimination prevails over otherwise valid impeachment grounds. D is incorrect. A conviction isn't necessary for prior criminal activity to be proper impeachment grounds.

In a federal investigation of a defendant for tax fraud, the grand jury seeks to obtain a letter written January 15 by the defendant to her attorney in which she stated: "Please prepare a deed giving my ranch to the university but, in order to get around the tax law, I want it back-dated to December 15." The attorney refuses to produce the letter on the ground of privilege. Production of the letter should be A: prohibited, because the statement is protected by the attorney-client privilege. B: prohibited, because the statement is protected by the client's privilege against self-incrimination. C: required, because the statement was in furtherance of crime or fraud. D: required, because the attorney-client privilege belongs to the client and can be claimed only by her.

C is correct. The attorney-client privilege will cover confidential communications made during a legal consultation between an attorney and client. However, if the services of the lawyer were sought or obtained to enable or aid anyone to commit or plan to commit a crime or a fraud, the privilege does not apply. The defendant's letter explicitly states that the deed she is seeking should be back-dated to avoid the tax law. Because the services sought by the defendant from her attorney were for the commission of tax fraud, the communication will not be privileged and the production of the letter should be required. A is incorrect. It ignores the crime-fraud exception to the attorney-client privilege. B is incorrect. The letter, although certainly incriminating, was not made to law enforcement officers in a custodialinterrogation. D is incorrect. The attorney can assert the privilege on behalf of the client.

A plaintiff is suing a doctor for medical malpractice occasioned by allegedly prescribing an incorrect medication, causing the plaintiff to undergo substantial hospitalization. When the doctor learned of the medication problem, she immediately offered to pay the plaintiff's hospital expenses. At trial, the plaintiff offers evidence of the doctor's offer to pay the costs of his hospitalization. The evidence of the doctor's offer is A: admissible as a nonhearsay statement of a party. B: admissible, although hearsay, as a statement against interest. C: inadmissible, because it is an offer to pay medical expenses. D: inadmissible, because it is an offer to compromise.

C is correct. The doctor's offer to the plaintiff is not within the definition of hearsay because it is an admission by a party-opponent. However, it is still not admissible because Federal Rule of Evidence (FRE) 409 states that the use of any offer or promise to pay the medical expenses of an injured party is inadmissible to prove liability for that injury. A is incorrect. While the doctor's offer is not hearsay, FRE 409 prevents the use of offers to pay medical expenses to prove liability. B is incorrect. As the defendant, the doctor's statement is not hearsay. Furthermore, there is no indication that the doctor is unavailable to testify, which is required for the statement against interest exception to apply. D is incorrect. The fact pattern does not indicate that the doctor asked for anything in return for the payment of the plaintiff's medical expenses. The doctor's statement is a naked offer to pay medical expenses.

A defendant was charged with the sale of narcotics. The federal prosecutor arranged with the defendant's wife for her to testify against her husband in exchange for leniency in her case. At trial, the prosecution calls the wife, who had been granted immunity from prosecution, to testify, among other things, that she saw her husband sell an ounce of heroin. Which of the following statements is most clearly correct in the federal courts? A: The defendant's wife cannot be called as a witness over her husband's objection. B: The defendant's wife can be called as a witness but cannot testify, over the defendant's objection, that she saw him sell heroin. C: The defendant's wife can refuse to be a witness against her husband. D: The defendant's wife can be required to be a witness and to testify that she saw her husband sell heroin.

C is correct. The wife's proposed testimony could lead to her invoking the adverse spousal privilege, a privilege against testifying against her husband. That privilege, however, only exists at the wife's discretion. The witness spouse, in this case the defendant's wife, alone has a privilege to refuse to testify adversely against her husband; she may be neither compelled to testify nor foreclosed from testifying. The choice is hers. A is incorrect. The adverse spousal privilege is the wife's, and the determination of whether to testify or not is herdecision to make. The defendant cannot object and force his wife not to testify. B is incorrect. The defendant's wife can testify, if she wishes, that she saw her husband sell heroin. Her testimonyabout her firsthand knowledge of the sale does not require that the wife disclose any privileged marital communication from her husband. She will testify as to what she saw. D is incorrect. The adverse spousal privilege still exists. If the wife wishes not to testify against her husband, she cannot be compelled, in federal court, to do so.

A defendant is charged with aggravated assault on a game warden. The defendant testified that, when he was confronted by the warden, who was armed and out of uniform, the defendant believed the warden was a robber and shot in self-defense. The state calls a witness to testify that a year earlier, he had seen the defendant shoot a man without provocation and thereafter falsely claimed self-defense. The witness's testimony is A: admissible, as evidence of the defendant's untruthfulness. B: admissible, as evidence that the defendant did not act in self-defense on this occasion. C: inadmissible, because it is improper character evidence. D: inadmissible, because it is irrelevant to the defense the defendant raised.

C is correct. The witness's testimony is improper extrinsic character evidence intended to prove the defendant's propensity for violence. The state cannot initiate evidence of the defendant's bad character merely to show that he is more likely to have committed the crime. A is incorrect. The prior shooting and self-defense claim is evidence of the defendant's violent character. Even if theevidence were relevant to the defendant's character for truthfulness, however, prior bad acts may not be proven by extrinsic evidence, only raised on cross-examination. B is incorrect. Evidence of a previous, unsuccessful self-defense claim introduced to establish that the defendant isfalsely claiming self-defense now is an improper use of character evidence; it is merely being used to show that the defendant is more likely to have committed the crime of which he is accused. D is incorrect. The evidence may in fact be relevant to the crime charged and/or the self-defense claim, but it is specifically excluded by the rules of evidence dealing with the character of the accused, as stated above.

Assume for purposes of this question only that the court concludes that Nurse Houlihan's statement "The sponge count came out wrong—there is one sponge unaccounted for" meets the definition of hearsay. Which one of the following statements reflects Walter's best argument that nurse Houlihan's statement is nonetheless excluded from hearsay? A: Nurse Houlihan was involved in the surgery and made her statement in furtherance of Dr. Burns's criminal malpractice. B: Nurse Houlihan's out-of-court statement was consistent with her in-court testimony. C: Dr. Burns's failure to respond to Nurse Houlihan's statement was so unreasonable that it supports the inference that Dr. Burns agreed with the statement. D: Nurse Houlihan's statement described an event made while or immediately after she perceived it.

C: Dr. Burns's failure to respond to Nurse Houlihan's statement was so unreasonable that it supports the inference that Dr. Burns agreed with the statement.

Don DeMarco was convicted an unregistered machine gun in violation of 26 U.S.C. § 5861(d). The government alleged he had altered a semiautomatic rifle so it would discharge more than one shot per trigger pull—the defining characteristic of a machine gun. The key question was whether the rifle would in fact rapid-fire. The government and Don each had their own experts test-fire it: In the government's test, the rifle did fire more than one shot per trigger pull, but when Don's expert (witnessed by two police officers) tested it, it didn't. Don's expert suggested the gun may have fired automatically in the government's test because the internal parts were dirty, worn, or defective. In response, the government attempted to introduce a photo of the rifle which, it argued, showed the rifle was in pristine condition and was neither dirty, worn, nor defective. The defendant objected, arguing that the photo was irrelevant because it showed nothing of the gun's interior and that the rifle in question only took up a small part of the photograph—the rest of the photo contained several other weapons including nine other guns and several knives belonging to Don's roommate. How should the court rule? A: SUSTAIN the objection, because the photo doesn't meet the relevancy standards under Rule 401—an exterior picture of the gun presents absolutely no probative evidence of the cleanliness of the gun's internal parts B: SUSTAIN the objection, because although the photo is relevant under Rule 401, the probative value doesn't outweigh the prejudicial effect. C: SUSTAIN the objection, because although the evidence is relevant under Rule 401, its prejudicial effect substantially outweighs its probative value. D: OVERRULE the objection, because the photo is relevant under Rule 401 and the probative value outweighs the prejudicial effect.

C: SUSTAIN the objection, because although the evidence is relevant under Rule 401, its prejudicial effect substantially outweighs its probative value.

Sheldon and Leonard set fire to a building to collect insurance money. They were both charged with arson and conspiracy to commit arson. Leonard cut a deal with the prosecution and agreed to testify against Sheldon. Sheldon and Leonard were tried separately. At Sheldon's trial, the prosecution seeks to admit Leonard's testimony that upon arriving at the building, Sheldon said to Leonard, "You brought the gasoline, right?" The prosecution offers this statement to prove that Leonard brought gasoline to the scene. Sheldon's attorney objects on the grounds of hearsay. Which of the following statements is correct? A: Sheldon's statement is not hearsay because it falls under the exclusion for an opposing party's statement made by the party's coconspirator in furtherance of the conspiracy. B: Leonard's testimony does not contain hearsay because questions can never constitute an assertion C: Sheldon's statement would meet the definition of hearsay if the judge determines that the question is sufficiently assertive. D: Sheldon's statement is not hearsay because Leonard is testifying as to Sheldon's statement in-court, so there is no out-of-court statement.

C: Sheldon's statement would meet the definition of hearsay if the judge determines that the question is sufficiently assertive.

A defendant was charged with conspiracy to possess cocaine with intent to distribute. While on bail with travel restricted to his home state, he purchased an airplane ticket to another country by using an alias. At trial, the prosecution seeks to introduce evidence of the defendant's ticket purchase. Should the court admit this evidence? A: No, because the evidence does not make any fact of consequence to the trial more or less probable than it would have been without the evidence. B: Yes, because the evidence is relevant both to show the defendant's consciousness of guilt and to show his motive to commit the crime. C: Yes, because the evidence is relevant to show the defendant's consciousness of guilt. D: Yes, because the evidence is relevant to show the defendant's motive to commit the crime.

C: Yes, because the evidence is relevant to show the defendant's consciousness of guilt. Federal Rule of Evidence (FRE) 401 defines relevance as evidence having "any tendency to make a fact [of consequence] more probable than it would be without the evidence." Evidence that the defendant tried to flee from prosecution—that he purchased an airplane ticket to another country using an alias—satisfies the definition of relevance, because it makes his guilt more probable than it would be without the evidence.

A plaintiff sued a defendant for injuries received when she fell down a stairway in the defendant's apartment building. The plaintiff, a guest in the building, alleged that she caught the heel of her shoe in a tear in the stair carpet. The plaintiff calls a tenant to testify that another resident had said to the tenant a week before the plaintiff's fall: "When I paid my rent this morning, I told the manager he had better fix that torn carpet." The resident's statement, reported by the tenant, is A: admissible, to prove that the carpet was defective. B: admissible, to prove that the defendant had notice of the defect. C: admissible, to prove both that the carpet was defective and that the defendant had notice of the defect. D: inadmissible, because it is hearsay not within any exception.

D is correct. The testimony by the tenant on the witness stand contains hearsay within hearsay ("multiple hearsay") and therefore, each hearsay statement must be separately admissible. Because the specific statement by the resident to the tenant is hearsay that does not fall within any exception, it renders the whole testimony inadmissible.

In a suit based on a will, the distribution of $1 million depends upon whether the wife survived her husband when both died in the crash of a small airplane. An applicable statute provides that, for purposes of distributing an estate after a common disaster, there is a rebuttable presumption that neither spouse survived the other. A witness has been called to testify that as she approached the plane she heard what she thought was a woman's voice saying, "I'm dying," although by the time the husband and wife were removed from the wreckage they were both dead. Is the witness's testimony admissible? A: No, because the matter is governed by the presumption that neither spouse survived the other. B: No, because the witness's testimony is too speculative to support a finding. C: Yes, because the hearsay rule does not apply to statements by decedents in actions to determine rights under a will. D: Yes, because it is relevant and not otherwise prohibited.

D is correct. The witness's testimony is relevant to the determination of whether the wife survived her husband, and no other rule of evidence bars its admission.

At the defendant's trial for stealing an automobile, the defendant called a character witness who testified that the defendant had an excellent reputation for honesty. In rebuttal, the prosecutor calls another witness to testify that he recently saw the defendant cheat on a college examination. The prosecution witness's testimony should be A: admitted, because the defendant has "opened the door" to the prosecutor's proof of bad character evidence. B: admitted, because the cheating involves "dishonesty or false statement." C: excluded, because it has no probative value on any issue in the case. D: excluded, because the defendant's cheating can be inquired into only on cross-examination of the defendant's witness.

D is correct. In rebutting a defendant's character witness, the prosecution may only ask about specific instances of the defendant's prior misconduct on cross-examination. A is incorrect. The prosecutor cannot use specific incidents of prior misconduct to rebut the character evidencetestimony given by the defendant's witness, except on cross-examination. B is incorrect. Even if cheating on an exam involved dishonesty or false statement, it is a specific act that may only beinquired into on cross-examination. C is incorrect. The prosecution's rebuttal witness would be improper character evidence, but not because the issue ofhonesty is not probative. In fact, the defendant made the issue relevant by introducing evidence of his reputation for honesty, and as such, the proffered testimony would be probative.

A passenger is suing a defendant for injuries suffered in the crash of a small airplane, alleging that the defendant had owned the plane and negligently failed to have it properly maintained. The defendant has asserted in defense that he never owned the plane or had any responsibility to maintain it. At trial, the passenger calls a witness to testify that the witness had sold to the defendant a liability insurance policy on the plane. The testimony of the witness is A: inadmissible, because the policy itself is required under the original document rule. B: inadmissible, because of the rule against proof of insurance where insurance is not itself at issue. C: admissible to show that the defendant had little motivation to invest money in maintenance of the airplane. D: admissible as some evidence of the defendant's ownership of or responsibility for the airplane.

D is correct. The witness's testimony that he had sold liability insurance on the plane to the defendant is admissible evidence to show the defendant's ownership or responsibility for the plane. It is relevant because the defendant has denied ownership. A is incorrect. The witness is not testifying to the content of the policy, only to its existence, and is not required tointroduce the policy itself before he can testify. B is incorrect. It misstates the standard for determining whether the witness's testimony is admissible. C is incorrect. The existence of the policy is admissible and relevant to show the defendant's ownership orresponsibility for the airplane, not his motivation to invest money in the airplane. It is the defendant's ownership of the plane that is at issue, and the witness's testimony is admissible as proof on that issue.

In a diversity action in federal court, the plaintiff seeks to have a drug abuse counselor from an employee assistance program testify about her communications with the defendant. The communications are privileged against disclosure under the relevant law of the state where the plaintiff's cause of action arose. What should govern the court's determination of the admissibility of the counselor's testimony? A: Federal common law as interpreted in the light of reason and experience. B: Federal statutory law. C: The court's sound discretion in the interest of justice. D: The state's privilege law.

D is correct. Under Federal Rule of Evidence (FRE) 501, state law governs privilege regarding a claim or defense for which state law supplies the rule of decision. A federal court sitting in diversity should look to the host state's choice of law rules when deciding which state's privilege law applies. A is incorrect. In general, the common law, as interpreted by US courts in the light of reason and experience, governs privilege; however, in a civil case, state law governs privilege regarding a claim or defense for which state law supplies the rule of decision. Fed. R. Evid. 501. B is incorrect. A federal court sitting in diversity applies state law according to the host state's choice of law rules. Fed.R. Evid. 501. C is incorrect. The admissibility of attorney-client testimony is governed by FRE 501 and requires a determination of the applicable state privilege law.

The plaintiff sued the defendant for damages for physical injuries allegedly caused by the defendant's violation of the federal civil rights law. The incident occurred wholly within a particular state but the case was tried in federal court. The state code says, "The common-law privileges are preserved intact in this state." At trial, the defendant called the plaintiff's physician to testify to confidential statements made to him by the plaintiff in furtherance of medical treatment for the injuries allegedly caused by the defendant. The plaintiff objects, claiming a physician-patient privilege. The court should apply A: state law and recognize the claim of privilege. B: federal law and recognize the claim of privilege. C: state law and reject the claim of privilege. D: federal law and reject the claim of privilege.

D is correct. When a case based on federal civil rights law is brought in federal court, the assertion of a privilege should be governed by federal common law. The federal courts do not recognize a physician-patient privilege. A is incorrect. The suit is in federal court, based on a violation of a federal civil rights law, so federal law is the applicable standard. B is incorrect. Federal courts do not recognize a physician-patient privilege. C is incorrect. As stated above, the suit has been brought in federal court, based on a violation of a federal civil right slaw, so federal law is the applicable standard.

The defendant is on trial for the crime of obstructing justice by concealing records subpoenaed May 1, in a government investigation. The government calls an attorney to testify that on May 3, the defendant asked him how to comply with the regulations regarding the transfer of records to a safe-deposit box in Mexico. The testimony of the attorney is A: privileged, because it relates to conduct outside the jurisdiction of the United States. B: privileged, because an attorney is required to keep the confidences of his clients. C: not privileged, provided the attorney knew of the concededly illegal purpose for which the advice was sought. D: not privileged, whether or not the attorney knew of the concededly illegal purpose for which the advice was sought.

D is correct. When a client seeks his attorney's advice in furtherance of planning to commit an illegal activity, the attorney-client privilege does not apply, regardless of whether the attorney was aware of the illegal activity. A is incorrect. Where the illegal conduct occurs is irrelevant to the non-applicability of the attorney-client privilege. B is incorrect. Although an attorney is required to keep his clients' confidences, this privilege does not apply to crimes or frauds being committed or planning to be committed. C is incorrect. The attorney's knowledge regarding the illegal purpose is irrelevant to whether the privilege will apply.

Personal injury action by P against D following an auto accident. P claims she suffered a back injury. To prove that she had sustained a back injury and was suffering pain as a result, P calls H, her husband to testify that almost every day for six months after the accident, P said to him, "My back is killing me." This statement is: A: Admissible as a party admission B: Admissible as an excited utterance C: Admissible because it is not hearsay—it is not being offered to prove the truth of the matter asserted D: Admissible under the exception for then-existing state of mind E: Admissible as a present-sense impression

D: Admissible under the exception for then-existing state of mind

Dr. Burns performed surgery on Walter. As was customary, nurse Houlihan kept track of how many sponges Dr. Burns placed inside Walter during the surgery to insure that Dr. Burns removed all the sponges by the end of the surgery. As Dr. Burns was finishing the surgery, nurse Houlihan told him, "The sponge count came out wrong—there is one sponge unaccounted for." Dr. Burns said nothing and completed the operation without searching for the sponge. Days later, Walter developed a severe infection. Dr. Pierce operated on Walter and discovered that Dr. Burns indeed had left a sponge inside Walter, and this sponge was the source of Walter's infection. Walter sued Dr. Burns for malpractice. To show that Dr. Burns was negligent for failing to check for the sponge, Walter offers nurse Houlihan's testimony about what she told Dr. Burns. Dr. Burns objects based on hearsay. Does nurse Houlihan's statement meet the definition of hearsay? A: YES, because it is an out-of-court statement used to prove the truth of the matter asserted B: YES, unless the statement was made for—and was reasonably pertinent to—medical diagnosis or treatment C: NO, because she testified as to her statement at trial, so it is an in-court statement rather than an out-of-court statement D: NO, because Walter is not using nurse Houlihan's statement for its truth but instead to show its effect on Dr. Burns

D: NO, because Walter is not using nurse Houlihan's statement for its truth but instead to show its effect on Dr. Burns

A university retained Samar as an independent-contractor public-relations representative. Her contract authorized her to help promote the university throughout the state by discussing the university's attributes, including its hiring and admissions practices, to help attract potential students and donors. During an interview with a radio reporter, Samar told the reporter, "Our school has a big problem concerning a lack of diversity." Harold heard Samar's comment on the radio. Harold had previously sued the university for racial discrimination, though he had not sued Samar individually. During the trial, to prove that the university had a big problem concerning a lack of diversity, Harold offers the radio reporter's testimony as to what Samar said during the interview. The university's attorney objects on the grounds of hearsay. How should the judge rule on this objection? A: OVERRULE the objection, because Samar's statement does not meet the definition of hearsay B: SUSTAIN the objection, because Samar was an independent contractor, not an employee C: SUSTAIN the objection, if Samar neither testified concerning the statement nor was subject to cross-examination concerning the statement D: OVERRULE the objection, because Samar's statement is excluded from the rule on hearsay

D: OVERRULE the objection, because Samar's statement is excluded from the rule on hearsay

John Richardson and Meg Anderson contracted for Meg's dog grooming company, "Perfect Pets," to provide grooming services to John's veterinary clinics across Houston. Within several months of the initial agreement, John began fielding complaints from the vet offices that dogs were being sloppily groomed. John sued Perfect Pets for breach of contract. Meg and John and their counsel met and agreed to settle the case. As a result of the settlement, Perfect Pets did not pay any monetary damages, but as the owner, Meg agreed to hire better groomers and to testify in John's trial against "Groomers R Us," the company that trained the groomers for Perfect Pets. At the trial between John and Groomers R Us, Meg testifies for John. After her testimony, counsel for Groomers R Us seeks to introduce evidence of the settlement agreement between John and Perfect Pets. John objects that the evidence is irrelevant. A: OVERRULE the objection, if the probative value of the evidence outweighs its prejudicial effect B: SUSTAIN the objection, because the evidence is irrelevant C: SUSTAIN the objection, because evidence of settlement negotiations is always inadmissible as a policy matter D: OVERRULE the objection, if the evidence is not offered to show liability

D: OVERRULE the objection, if the evidence is not offered to show liability

Jacqueline Morrow worked for MD Anderson for 20 years. In the course of her employment, Jacqueline embezzled over $60,000 from the hospital. After discovering the missing funds, MD Anderson informed the Harris County District Attorney's Office. Following an investigation, Jacqueline was indicted for the crime. At the time of the indictment, Jacqueline was 55 years old and extremely scared about the possibility of serving prison time. Jacqueline went to her former boss at MD Anderson to discuss the charges against her and try to negotiate a plea for a reduced charge. Jacqueline admitted guilt, but explained that she only did it to ensure her son could attend college. Jacqueline's former boss refused to say anything and ordered Jacqueline off of the property. The DA seeks to introduce Jacqueline's statements to her former boss into evidence at trial. Defense counsel objects. How should the court rule? A: SUSTAIN the objection, as a statement made during plea negotiations. B: SUSTAIN the objection, because the prejudicial effect of the statement outweighs its probative value. C: SUSTAIN the objection, because the evidence is not relevant under 401. D: OVERRULE the objection.

D: OVERRULE the objection.

David has been charged with possession of a controlled substance with intent to distribute. At trial, David seeks to stipulate to the fact that he possessed 2.5 grams of cocaine, focusing his defense on the evidence regarding intent to distribute. The prosecution argues against the stipulation, and wants to admit evidence of the facts relating to possession. How should the court rule? A: Allow the stipulation. B: Reject the stipulation if the probative value outweighs the prejudicial effect. C: Allow the stipulation if the prejudicial effect of the evidence would outweigh the probative value of the evidence. D: Reject the stipulation.

D: Reject the stipulation.

Bob is being prosecuted for bank robbery. Alice, an alleged accomplice, told police while under interrogation that she was the mastermind of the crime but that Bob was also involved. Alice died while in custody. Which of the following is most accurate? A: Alice's statement is admissible as a declaration against interest. B: Alice's statement is not hearsay. C: Alice's statement is admissible as a party admission because of the co-conspirator rule. D: The admission of Alice's statement would violate the Confrontation Clause.

D: The admission of Alice's statement would violate the Confrontation Clause.

ESSAY: Six months ago, a woman was taken to a hospital following what she alleged was a sexual assault by a man during a fraternity party. The woman and the man were both seniors attending the college where the party was held. At the time of the alleged assault, the hospital's policy required that "in all cases of alleged or suspected sexual assault, non-emergency patients must be interviewed by a victim counselor before receiving medical treatment." The woman was deemed a non-emergency patient and was told to wait in the waiting room to see a victim counselor. Three hours later, the victim counselor finally interviewed the woman. Thereafter, hospital personnel treated the woman for her injuries and sent her home. There was no contact between the woman and the man until one week later, when the man sent the woman a text message on her cell phone. The text message said, "If you are upset about what happened, I can send you a check for $10,000 to help you forget the whole thing. I can also pay any medical expenses." The woman did not respond. Four months after the alleged assault, the woman contacted a lawyer and filed a civil action against the man and the hospital. She sought damages from the man for physical injuries resulting from the alleged assault. She also sought damages from the man for psychological injuries. According to the woman, these injuries were especially traumatic because of her belief in sexual abstinence before marriage and her lack of prior sexual experience. She sought damages from the hospital for exacerbating her injuries by negligently delaying her medical treatment. The man filed an answer admitting that he had had sexual relations with the woman but asserting that they were consensual. In its answer, the hospital denied that its conduct had exacerbated the woman's injuries. Immediately after filing its answer, the hospital contacted the woman and offered to settle the claim for $5,000. The woman refused the hospital's offer. Five weeks after the woman filed her suit, the hospital changed its policy on dealing with sexual assault victims to provide that "in all cases of alleged or suspected sexual assault, immediate medical care will be provided to emergency and non-emergency patients." The woman's suit against the man and the hospital is now set for trial. The following properly filed motions are before the court: The hospital's motion to exclude evidence of its new policy providing immediate medical treatment to emergency and non-emergency patients in all cases of alleged or suspected sexual assault. The hospital's motion to exclude evidence of its offer to settle with the woman. The man's motion to exclude evidence of a) his offer to pay the woman $10,000. b) his offer to pay the woman's medical expenses. The rules of evidence in this jurisdiction are identical to the Federal Rules of Evidence. Assume that there are no hearsay issues with any of these pieces of evidence. How should the court rule on each of these motions? Explain.

ESSAY


Kaugnay na mga set ng pag-aaral

Harding Chapter 59 Alz/Dementia Evolve.Elsevier

View Set

Quiz Questions for Honors Biology || Quarter 1

View Set

EMT - Chapter 34: Pediatric Emergencies

View Set

Articles of Confederation/ Shays Rebellion - Ss - 9/19/19 - Period 3 - Sherene Al-Salimi

View Set

M & B Banking and MGMT of FI chapter 10

View Set

Ch. 1 Anatomy and Physiology for Emergency Care

View Set

Chapter 1 Leading, Managing, following

View Set